huang bought 12 boxes of floor tiles that weight 288 pounds each. how much is this in kilograms

Answers

Answer 1

[tex]in \: pounds \\ 12 \times 288 = 3456 \: pounds[/tex]

[tex]1 \: kilo = 2.2046 \: pounds[/tex]

[tex]x \: kilos = 3456 \: pounds[/tex]

[tex]x = \frac{3456}{2.2046} = 1567.61 \: kilos[/tex]


Related Questions

While driving, Carl notices that his odometer reads 25,952 miles, which happens to be a palindrome. He thought this was pretty rare, but 2.5 hours later, his odometer reads as 36,563 miles, anoher palindrome. What was Carl's average speed during those 2.5 hours

Answers

Carl's average speed was 4244 miles/hr during those 2.5 hours

First, the odometer showed palindromic no. 25952 miles

after 2.5 hours odometer showed the next palindromic no. is 36563.

hence the distance covered in 2.5 hours = 36563 - 25952 = 110611 miles.

Now, we have to find the average speed with the distance covered 110611 miles in a particular time of 2.5 hours.

Using formula,

average speed = total distance / total time

                    [tex]\frac{10611}{2.5}[/tex] =4244 miles/hour is the average speed

Hence, Carl's average speed was 4244 miles/hr during those 2.5 hours

Learn more about speed and time  here https://brainly.com/question/6504879

#SPJ4

What is the probability that the manufacturing unit has carbon emission beyond the permissible emission level and the test predicts this?

Answers

The probability that the manufacturing unit has carbon emissions beyond the permissible emission level is 0.2975

Given that the probability that carbon emissions from the company’s factory exceed the permissible level is 35% and the accuracy of the test is 85%.

The possibility of an event or outcome happening contingent on the occurrence of a prior event or outcome is known as conditional probability. The probability of the prior event is multiplied by the current likelihood of the subsequent, or conditional, occurrence to determine the conditional probability.

Event A: The given Carbon emission beyond to the given permissible emission level.

Event B: Test predicts this.

To get the probability of this problem, first we need to divide by 100 the given values.

A=35/100

A=0.35

The probability of event A is P(A)=0.35

And the probability is P(B|A)=85/100=0.85

Now, we will use the conditional probability formula, we get

P(B|A)=P(A∩B)/P(A)

0.85=P(A∩B)/0.35

P(A∩B)=0.85×0.35

P(A∩B)=0.2975

Hence, the probability that the manufacturing unit has carbon emissions beyond the permissible emission level and the test predicts is0.2975.

Learn more about conditional probability from here brainly.com/question/12985746

#SPJ4

Find the area of the shaded region. (Object is not drawn to scale.) GEOMETRY PLS HELPPPP

Answers

Answer:

101.5 cm

Step-by-step explanation:

You find the area of the rectangle and then the area of the triangle. Then, you subtract the area of the rectangle and the area of triangle.

Which expression is equivalent to 3x2 + 7x + 4?
3(x2 + 7x + 4) – 14x - 8
3x2 + 7x + 4 – 14x
3x2 + 7x + 4 – 8
3x2 + 7x

Answers

Answer:

a.) 3(x^2 + 7x + 4) – 14x - 8

Step-by-step explanation:

a.) 3(x^2 + 7x + 4) – 14x - 8

3x^2 + 21x + 12 - 14x - 8

3x^2 + 7x + 4

b.) 3x^2 + 7x + 4 – 14x

3x^2 -7x + 4

c.) 3x^2 + 7x + 4 – 8

3x^2 + 7x - 4

d.) 3x^2 + 7x

Already simplified

please help me on this question. im not the best at geometry

Answers

Answer: 48

Step-by-step explanation:

By the geometric mean theorem,

[tex]\frac{x}{36}=\frac{64}{x}\\\\x^{2}=36 \cdot 64\\\\x=\sqrt{36 \cdot 64}\\\\x=\sqrt{36} \sqrt{64}=48[/tex]

Giving 100 points.

Noah manages a buffet at a local restaurant. He charges $10 for the buffet. On average, 16 customers choose the buffet as their meal every hour. After surveying several customers, Noah has determined that for every $1 increase in the cost of the buffet, the average number of customers who select the buffet will decrease by 2 per hour. The restaurant owner wants the buffet to maintain a minimum revenue of $130 per hour.


Noah wants to model this situation with an inequality and use the model to help him make the best pricing decisions.


1. Write two expressions for this situation, one representing the cost per customer and the other representing the average number of customers. Assume that x represents the number of $1 increases in the cost of the buffet.


2. To calculate the hourly revenue from the buffet after x $1 increases, multiply the price paid by each customer and the average number of customers per hour. Create an inequality in standard form that represents the restaurant owner’s desired revenue.


3. Which possible buffet prices could Noah could charge and still maintain the restaurant owner’s revenue requirements?

Answers

Answer:

1.   Cost per customer:  10 + x

     Average number of customers:  16 - 2x

[tex]\textsf{2.} \quad -2x^2-4x+160\geq 130[/tex]

3.    $10, $11, $12 and $13

Step-by-step explanation:

Given information:

$10 = cost of buffet per customer16 customers choose the buffet per hourEvery $1 increase in the cost of the buffet = loss of 2 customers per hour$130 = minimum revenue needed per hour

Let x = the number of $1 increases in the cost of the buffet

Part 1

Cost per customer:  10 + x

Average number of customers:  16 - 2x

Part 2

The cost per customer multiplied by the number of customers needs to be at least $130.  Therefore, we can use the expressions found in part 1 to write the inequality:

[tex](10 + x)(16 - 2x)\geq 130[/tex]

[tex]\implies 160-20x+16x-2x^2\geq 130[/tex]

[tex]\implies -2x^2-4x+160\geq 130[/tex]

Part 3

To determine the possible buffet prices that Noah could charge and still maintain the restaurant owner's revenue requirements, solve the inequality:

[tex]\implies -2x^2-4x+160\geq 130[/tex]

[tex]\implies -2x^2-4x+30\geq 0[/tex]

[tex]\implies -2(x^2+2x-15)\geq 0[/tex]

[tex]\implies x^2+2x-15\leq 0[/tex]

[tex]\implies (x-3)(x+5)\leq 0[/tex]

Find the roots by equating to zero:

[tex]\implies (x-3)(x+5)=0[/tex]

[tex]x-3=0 \implies x=3[/tex]

[tex]x+5=0 \implies x=-5[/tex]

Therefore, the roots are x = 3 and x = -5.

Test the roots by choosing a value between the roots and substituting it into the original inequality:

[tex]\textsf{At }x=2: \quad -2(2)^2-4(2)+160=144[/tex]

As 144 ≥ 130, the solution to the inequality is between the roots:  

-5 ≤ x ≤ 3

To find the range of possible buffet prices Noah could charge and still maintain a minimum revenue of $130, substitute x = 0 and x = 3 into the expression for "cost per customer.  

[Please note that we cannot use the negative values of the possible values of x since the question only tells us information about the change in average customers per hour considering an increase in cost.  It does not confirm that if the cost is reduced (less than $10) the number of customers increases per hour.]

Cost per customer:  

[tex]x =0 \implies 10 + 0=\$10[/tex]

[tex]x=3 \implies 10+3=\$13[/tex]

Therefore, the possible buffet prices Noah could charge are:

$10, $11, $12 and $13.

8. What is the domain of [tex]\frac{x}{x^{2} +20x+75}[/tex] ? Hint: try factoring the polynomial.

9. What is the domain and range of [tex]\sqrt{13x-7}+1[/tex] ?

Please hurry! I really need help with this!

Answers

Answer:

8.  Domain: (-∞, -15) ∪ (-15, -5) ∪ (-5, ∞)

9.  Domain: [7/13, ∞)

    Range: [1, ∞)

Step-by-step explanation:

Question 8

Given rational function:

[tex]f(x)=\dfrac{x}{x^2+20x+75}[/tex]

Factor the denominator of the given rational function:

[tex]\implies x^2+20x+75[/tex]

[tex]\implies x^2+5x+15x+75[/tex]

[tex]\implies x(x+5)+15(x+5)[/tex]

[tex]\implies (x+15)(x+5)[/tex]

Therefore:

[tex]f(x)=\dfrac{x}{(x+15)(x+5)}[/tex]

Asymptote: a line that the curve gets infinitely close to, but never touches.

The function is undefined when the denominator equals zero:

[tex]x+15=0 \implies x=-15[/tex]

[tex]x+5=0 \implies x=-5[/tex]

Therefore, there are vertical asymptotes at x = -15 and x = -5.

Domain: set of all possible input values (x-values)

Therefore, the domain of the given rational function is:

(-∞, -15) ∪ (-15, -5) ∪ (-5, ∞)

---------------------------------------------------------------------------------

Question 9

Given function:

[tex]f(x)=\sqrt{13x-7}+1[/tex]

Domain: set of all possible input values (x-values)

As the square root of a negative number is undefined:

[tex]\implies 13x-7\geq 0[/tex]

[tex]\implies 13x\geq 7[/tex]

[tex]\implies x\geq \dfrac{7}{13}[/tex]

Therefore, the domain of the given function is:

[tex]\left[\dfrac{7}{13},\infty\right)[/tex]

Range: set of all possible output values (y-values)

[tex]\textsf{As }\:\sqrt{13x-7}\geq 0[/tex]

[tex]\implies \sqrt{13x-7}+1\geq 1[/tex]

Therefore, the range of the given function is:

[1, ∞)

can someone help me solve this

Answers

Answer:

Step-by-step explanation:

Answer:

26.9 ft

Step-by-step explanation:

So this answer is going to require one of the trigonometric functions, which is the ratio of sides of a triangle. So I attached a diagram to my answer, which is depicting the given information and hopefully this should be able to help a bit.

Anyways, we know the hypotenuse, an angle, and the opposite side of the angle. There is a trigonometric function which is defined as: [tex]sin(\theta)=\frac{opposite}{hypotenuse}[/tex]. We can use the sine function to solve for the opposite side. So let's plug in known values into this equation:

Known Values: theta=85 degrees, hypotenuse = 27 ft

[tex]sin(85)=\frac{opposite}{27 ft}[/tex]

Multiply both sides by 27 ft]

[tex]sin(85)*27ft=opposite[/tex]

Now you can approximate the value of sin(85) using your calculator, and make sure it's in degree mode not radian mode.

[tex]0.996194698*27ft\approx opposite[/tex]

Simplify

[tex]26.8973 ft\approx opposite[/tex]

Rounding this to one decimal place makes it

[tex]26.9 ft\approx opposite[/tex]

Find the value of each variable in the diagram given that a ∥ b.

Can you also explain please? Thank you!

Answers

The values are as follows:

z=50

v= 150

x= 100

y=15

w= 50

What is parallel lines?

Parallel lines are lines in a plane that are always the same distance apart.

z+130= 180   (Linear pair)

z=50

v+ 30 = 180  (Linear pair)

v= 150

x= 180- 30 - 50  (angle sum property)

x= 100

2y=30   (alternate interior angle)

y=15

w+x+2y=180  (Linear pair)

w+ 100 + 30 = 180

w= 50

Learn more parallel lines here:

https://brainly.com/question/16701300

#SPJ1

Nina, Shanti and Belle run a 1000 m race at a constant speed. When Nina crossed the inish line irst,
she was 200 m ahead of Shanti and 400 m ahead of Belle. When Shanti crossed the inish line, how far
ahead of Belle was she?

Answers

When Shanti had crossed the finish line, she was a distance of 250 m ahead of Belle

How to calculate Constant Speed?

We are given;

Length of Race = 1000 m

When nina crossed the finish line, she was 200 m ahead of Shanti and 400 m ahead of Belle.

Thus;

Shanti had run 4/5 of the race when Nina finished.

So, if Nina's time was x, then by variation, we can say that;

Time it took Shanti to finish the race = 5/4x

So, Belle had run 5/4 * 600 = 750 meters when Shanti finished.

Thus, means Shanti was; 1000 - 750 = 250m ahead of Belle.

Read more about Constant Speed at; https://brainly.com/question/2681210

#SPJ1

Sherry Lebito bought 500 shares of stock at a quoted price of 83 1/4 What was the total purchase price?
$


A year later Sherry sold her stock at a quoted price of 95 1/2.
(b) Did she have a capital gain or a capital loss?


(c) How much of a gain or loss did she experience?
$

Answers

The total purchase price is $41,625.00

Capital gain was recorded

The capital gain is $41,625.00

What is total initial initial investment?

Was a capital gain or loss made?

What is the dollar value of gain or loss recorded?

Cost per share=$83.25(note 1/4=0.25)

Total purchase price=purchase price per share*number of shares

Total purchase price=$83.25*500

Total purchase price=$41,625.00

The fact that the price at which Sherry sold the stocks a year later is more than the initial purchase price of $83.25, means that a capital gain was recorded, in other words, she bought low and sold high.

The capital gain can be computed as the total selling price(selling price per stock multiplied by the number of shares sold) minus the total purchase price.

total selling price=$95.50(i.e. 1/2=0.500

total selling price=$95.50*500

total selling price=$47,750.00

capital gain=$47,750.00-$41,625.00

capital gain=$6,125.00

https://brainly.com/question/1259480

#SPJ1

Miss Goh bought 6 blouses, 2 skirts and 2 dresses for $560. A blouse and a skirt cost $107. A blouse and a dress cost $134. What was the cost of a dress?

Answers

Answer:

$95 FOR A DRESS

Step-by-step explanation:

Let's rearrange the given:

  She bought a blouse and skirt for two, a blouse and a dress for two, and a blouse for two.

     A blouse and a skirt cost $107

     A blouse and a dress cost $134

     blouse cost x dollars

The equation:

     2($107) + 2($134) + 2x = $560

SOLUTION:

   $214 + $268 + 2x = $560

   2x = $560 - $214 - $268

   2x = $78

   x = $39 for a single blouse

FIND THE COST OF DRESS:

   A blouse and a dress cost $134 - $39 for a single blouse

      $134 - $39 = $95

$95 IS THE COST OF DRESS

   

The graph below shows a company's profit f(x), in dollars, depending on the price of pens x, in dollars, sold by the company:

Graph of quadratic function f of x having x intercepts at ordered pairs 0, 0 and 6, 0. The vertex is at 3, 120.

Part A: What do the x-intercepts and maximum value of the graph represent? What are the intervals where the function is increasing and decreasing, and what do they represent about the sale and profit? (4 points)

Part B: What is an approximate average rate of change of the graph from x = 3 to x = 5, and what does this rate represent? (3 points)

Part C: Describe the constraints of the domain. (3 points)

Answers

The answers to the various part as well as its reasons are given below

Part A:The x-intercepts shows a zero profit.The maximum value of the graph tells  or depict the maximum profit.The function is one that goes up or increases upward until it reach the vertex and then it falls or decreases after it.This implies that the profit goes up as it reaches the peak at the vertex and it goes down after the vertex up until it gets to zero.The  profits are negative as seen on the left of the first zero and on the right of the second zero.

Part B:

An approximate average rate of change of the graph from x = 3 to x = 5, shows the reduction in profit from  3 to 5.

Part C:

Based on the above,  the domain is one that is held back or constrained by x = 0 .

We are compelled at x = 6 due to the fact that we have to maneuver a negative profit.

Learn more about quadratic function from

https://brainly.com/question/25841119

#SPJ1

Manuel is blowing up balloons for a party at his house .he blows up 30 balloons in 10 minutes.how many balloons does he blow up in one minute ?

Answers

Answer:

it is 3 i think

Step-by-step explanatplease mark brainlest

I can walk at 6km/hr and run at 15km/hr. On a journey I spend as long walking as I do running. If I had walked twice as long on the journey it would have taken me six minutes longer find how far I ran​

Answers

He ran for 12 km in his journey.

What is an Equation ?

An equation is formed when two algebraic expressions are equated by an equal sign.

It is given that

Speed of Walking =6km/hr  

Speed of Running = 15km/hr

Let he walks and run x hours ( as given)

he covered distance = 21 x km

The total time taken in the journey is 2x

for the same distance

It is also given that

if for the total journey

if he walks walked twice as long on the journey

He walked 6x in the journey so new distance that he walked is  12x  , then the total journey time = 6 minutes longer

Converting speed from km / hr to km/ min

1 km / hr = 60 km/ min

and forming an equation with the given data

12x * 60 /6 + 9x *60/15 = 2x * 60 + 6

120x + 36x = 120x +6

150x = 120

x = 120/150

x = 0.8 hours.

Therefore he spent 0.8 hours walking and 0.8 hours running and He ran 15 * 0.8 = 12 km

To know more about Equation

https://brainly.com/question/10413253

#SPJ1

From the diagram below, given the side lengths marked, and if we know that < C is congruent to < E, we can say that ___.



Select one:

a.
the two triangles are similar by SAS


b.
the two triangles are not similar


c.
the two triangles are congruent


d.
the two triangles are similar by AA

Answers

Answer:

d) the two triangles are similar by AA

Step-by-step explanation:

A car rental company offers two plans for renting a car. Plan A: $30 per day and 18 cents per mile Plan B: $66 per day with free unlimited mileage How many miles would you need to drive for plan B to save you money

Answers

Answer:

200 miles

Step-by-step explanation:

Set up a cost equation for each plan using x for miles.

Set the equations equal.

Solve for x, the miles.

For a 1 day rental.

Plan A:

C = 0.18x + 30

Plan B:

C = 66

0.18x + 30 = 66

0.18x = 36

x = 36/0.18

x = 200

Answer: 200 miles

A fence is to be built to enclose a rectangular area of 200 square feet. The fence along three sides is to be made of material that costs 3 dollars per foot, and the material for the fourth side costs 12 dollars per foot. Find the dimensions of the enclosure that is most economical to construct.

Answers

y = 220/ ((10/3) * √22)

It is given that area is 200 square feet. Cost is 3 dollar per foot. Fourth side costs 12 dollars per foot.

This f(x,y) needs to represent the cost of the fence so we can look at each side's price. We can choose the fourth side to be along the x axis and be represented by 13x. The other sides therefore must be represented by 5y, 5y, and 5x.

So, our cost, f(x,y) = 13x + 5x + 5y + 5y = 18x + 10y

Our constraint is xy = 220 as defined by the area of a rectangle.

We can then take our constraint to be put in terms of exclusively x, giving us y = 220/x

Plugging this into our cost, the thing we are minimizing, we get f(x) = 18x + 2200/x

In order to find the minimum we use the first derivative test, taking f'(x) and finding the critical points.

f'(x) = 18 - 2200/x2

Setting this equation to be equal to 0 we find that x = ±√2200/18 . But the negative answer doesn't make sense because distance cannot be negative so we throw it out.

x = (10/3) * √22

We must verify that this is a minimum by confirming the following:

If x < (10/3) * √22, f'(x) < 0. So, f(x) is decreasing when x < (10/3) * √22.

If x > (10/3) * √22, f'(x) > 0. So, f(x) is increasing when x > (10/3) * √22.

Thus, we have guaranteed that (10/3) * √22 is the x dimension. Now we plug in this value in our original equation to find y and that is our y dimension. So, y = 220/ ((10/3) * √22)

Learn more about word problems here: brainly.com/question/21405634

#SPJ4

a chemical company makes two brands of antifreeze the first brand is 20% pure antifreeze and the second brand is 45% pure antifreeze in order to obtain 70 gallons of the mixture that contains 30% antifreeze how many gallons of each brand of antifreeze must be used ​

Answers

Answer:

first brand: 42 gallons

second brand: 28 gallons

Step-by-step explanation:

So you know that the first brand is 20% antifreeze, this means that if you have x gallons then 0.20x is pure antifreeze. The same thing can be applied for the second brand except the coefficient is 0.45 and I'll just say the variable is y. If you haven't noticed, the coefficients are simply the percentages of antifreeze, but in decimal form. Anyways, since it's asking for a mixture that's 30% antifreeze and is 70 gallons that means 0.30(70) gallons is pure antifreeze. This simplifies to 21 gallons. So if we take the pure antifreeze of both mixtures we get the following equation:

[tex]21 = 0.2x + 0.45y[/tex]

the x represents how many gallons of the first brand, and the y represents how many gallons of the second brand. Since that's what it represents we also get the equation:

[tex]70=x+y[/tex]

In this equation there aren't any coefficients, since x and y represent the whole amount. the 0.2x and 0.45 simply represent how much is pure antifreeze.

So to solve this systems of equation we can solve for y or x, either works, and then substitute it into the other equation. So in this example I'll solve for y

Original equation:

[tex]70=x+y[/tex]

Subtract x from both sides

[tex]70-x=y[/tex]

So now we can substitute this into the pure antifreeze question

[tex]21 = 0.2x + 0.45y \implies21=0.2x+0.45(70-x)[/tex]

So now we can solve for x since it's the only variable in this equation

[tex]21=0.2x+31.5-0.45x[/tex]

Simplify

[tex]21=-0.25x+31.5[/tex]

Subtract 31.5 from both sides

[tex]-10.5=-0.25x[/tex]

Now divide both sides by -0.25

[tex]42=x[/tex]

So this means we have to use 42 gallons of the first brand, now we can substitute this into either equation to solve for y, but it's much easier to use the total gallon equation:

[tex]x+y=70\implies 42+y=70[/tex]

Subtract 42 from both sides

[tex]y=28[/tex]

So this means we need to use 42 gallons of the first brand and 28 gallons of the second brand

12 A pilot needs to know the maximum height an aircraft can fly at. The cabin has been tested and is safe to a height of 15679 m. Round this height appropriately: a) to a whole number of kilometres b) to a whole 100 m.please help me​

Answers

15.6 kilometers rounded down because it is not safe to go above what has been approved

The length of a rectangle is 7 centimeters less than three times its width. Its area is 40 square centimeters. Find the dimensions of the rectangle.

Answers

If the area of the rectangle is 40 square centimeters and length is 7 cm less than 3 times the breadth then the length will be 8 cm and breadth will be 5 cm.

Given area of the rectangle is 40 square centimeters and length is 7 centimeters less than 3 times the breadth.

let the breadth of rectangle be x.

According to question

Length=3x-7

Breadth=x

Area of  rectangle=length* breadth

=[tex](3x-7)x[/tex]

=[tex]3x^{2} -7x[/tex]

Area is given as 40 square centimeters.

40=[tex]3x^{2} -7x[/tex]

[tex]3x^{2} -7x-40=0[/tex]

By factorization

[tex]3x^{2} -15x+8x-40=0[/tex]

3x(x-5)+8(x-5)=0

(3x+8)(x-5)=0

3x+8=0

x=-8/3

x-5=0

x=5

We should neglect x=-8/3 because side cannot be negative. So we have taken x=5.

Put x=5 in 3x-7 to get length

length=3(5)-7

=15-7

=8

Hence the length will be 8 cm and breadth will be 5 cm.

Learn more about area at https://brainly.com/question/25965491

#SPJ4

find the root of the function f(x)= 1/(1/3)

Answers

Answer:

Hello, look at the picture

Drag the tiles to the correct boxes to complete the pairs.
Match each question with its answer. The answers are in the form (quotient)r (remainder).

Answers

The number of books on each shelf is 11 remainder 11

The toffees left over is 22 r 4

The oranges that would remain is 11 r30

The pages left to read is 22 r 16

What are the matched questions and answers?

Division is the process of grouping a number into equal parts using another number. The sign used to denote division is ÷.

Number of books on each shelf = 3542 / 321 = 11 remainder 11

The toffees left over = 246 / 11 = 22 r 4

The oranges that would remain: 1702 / 152 = 11 r30

The pages left to read = 676 / 30 = 22 r 16

To learn more about division, please check: https://brainly.com/question/13281206

#SPJ1

3. It snowed 80 inches over 10 days. What is the rate? What about the unit rate?

Answers

It snowed 8 inches of snow per day.

What is the smallest sample size that guarantees that the margin of error is less than 1% when constructing a 97% confidence interval for a population proportion

Answers

In order to get 97% confidence level, with confidence interval of

+/- 1%, and standard deviation of 0.5 our sample size should be 11,772 samples

A confidence interval for a population mean, when the population standard deviation is known based on the conclusion of the Central Limit Theorem that the sampling distribution of the sample means follow an approximately normal distribution.

Accordingly, the Necessary Sample Size is calculated as follows:

Necessary Sample Size = [tex](Z-score)^{2} . StdDev*(1-StdDev) / (Margin Of Error)^{2}[/tex]

Given:-

At  97% confidence level Z-score = 2.17 Assuming standard deviation = 0.5, margin of error (confidence interval) of +/- 1%.

Substituting in the given formula we get

Necessary Sample Size =

= ((2.17)² x 0.5(0.5)) / (0.01)²

= (4.7089x 0.25) / .0001

= 1.177225 / 0.0001

= 11,772.25

Hence in order to get 97% confidence level, with confidence interval of

+/- 1%,  and standard deviation of 0.5 our sample size should be 11,772

Learn more about sample size here :

https://brainly.com/question/10068670

#SPJ4

The sum of two numbers is 724. one of the numbers is 217. what is the other number?

Answers

Answer:

507

Step-by-step explanation:

For the Algebraic Expression,

let the two numbers be X and y

lets assume x = 217

From the question,

217 + y = 724

y = 724-217

y = 507

Therefore, the other number is 507

find the area of a circle whose radius is 'x', unit. ​

Answers

3.14 x² units

Step-by-step explanation:

Radius = x units. ----(given)

π = 3.14

Area of cirlce = πr²

= 3.14 * x²

= 3.14 x² units.

Hope it helps you!!

Answer:

area = πx² square units

Step-by-step explanation:

Formula for the area of a circle:

area = πr²,

where r = radius.

In this problem, the radius is x, so we substitute r in the formula with x.

area = πx² square units


Mixed Practice
In the following exercises, solve.
358. The length of one leg of a right triangle is three more than the other leg. If the hypotenuse is 15, find the lengths of the two legs.

Answers

Answer:

9 units and 12 units to nearest hundredth.

Step-by-step explanation:

Let one leg be x units long  then the other = x+3 units.

By Pythagoras:

x^2 + (x+3)^2 = 15^2 = 225

2x^2 + 6x + 9 = 225

2x^2 + 6x - 216 =0

x^2 + 3x - 108 = 0

(x - 9)(x + 12) = 0

x = 9

So the other 2 legs are 9 and 12.

O Here is a list of numbers:
19, 4, 7, 2, 10, 5, 19, 2, 12
State the median.

Answers

Answer:

10

Step-by-step explanation:

median is the middle number of the list of numbers.

thereby counting through, the middle number which is the fifth number is 10

Answer:

7

Step-by-step explanation:

A manager measured the number of goods, y, that his company produced in x hours. The company produces goods at a rate of 5 per hour. At hour 9, the company had produced 45 goods.

Which equation, in point-slope form, correctly represents the goods produced by the company after x hours?

Answers

The equation, in point-slope form is (y - 45) = 5(x - 9) option first is correct.

What is linear equation?

It is defined as the relation between two variables, if we plot the graph of the linear equation we will get a straight line.

If in the linear equation, one variable is present, then the equation is known as the linear equation in one variable.

The question is incomplete.

The complete question is in the picture, please refer to the attached picture.

A manager measured the number of goods, y, that his company produced in x hours.

The slope:

m = 5

From the problem:

The point (9, 45) will satisfy the line.

(y - 45) = 5(x - 9)

Thus, the equation, in point-slope form is (y - 45) = 5(x - 9) option first is correct.

Learn more about the linear equation here:

brainly.com/question/11897796

#SPJ1

Other Questions
At what point in American history were these threeamendments ratified?before the Civil Warduring Presidential Reconstructionduring Congressional Reconstructionat the end of the Reconstruction era I need help with number 7 and 8 and 9! Can someone help me with this please? How does labor relate to the scarcity problem In general, readers today have a better understanding of science than readersin 1898. How would the tone of the passage's narrator have affected readersin 1898 versus readers today? Determine the equation, in vertex form, of a parabola that has been:(a) reflected in the x-axis and horizontally shifted right 4 units.(b) vertically compressed by a factor of 1/4 and vertically translated 5 units down(c) vertically stretched by a factor of 4, translated 3 units left and 2 units up A telegraph pole stands on a line joining two points P and Q on the ground.The telegraph pole is 9 m tall. The angle of depression of the top of the pole to point Pis 4. The angle of depression of the top of the pole to point Q is 7. What is thehorizontal distance between P and Q. How do you think human life has changed over the past 1,00 0 years? igh levels of cortisol would normally provide negative feedback to the _______. High levels of cortisol would normally provide negative feedback to the _______. hypothalamus the posterior pituitary and the hypothalamus anterior pituitary posterior pituitary the anterior pituitary and the hypothalamus Have you (t) seen the new movie?Has visto la nueva pelcula?Ha visto la nueva pelcula?Han visitado la nueva pelcula?Hemos visto la nueva pelcula? An advice or advice? How many outfits can be made with 40 shirts and 15 pairs of pants The mean annual cost of an automotive insurance policy is normally distributed with a mean of $1140 and standard deviation of $310.a. What is the probability that a random sample of 16 policyholders will have a mean insurance policy cost between $1000 and $1250?Round your z value(s) to two decimal places. Do not round any other intermediate calculations. Round your answer to four decimal places.Probability = b. What is the probability that a random sample of 16 policyholders will have a mean insurance policy cost which exceeds $1250?Round your z value(s) to two decimal places. Do not round any other intermediate calculations. Round your answer to four decimal places.Probability = c. What is the probability that a random sample of 16 policyholders will have a mean insurance policy cost below $1220?Round your z value(s) to two decimal places. Do not round any other intermediate calculations. Round your answer to four decimal places.Probability = what is the following product? 10 10 A)10 B) 10/10 C) 1010D) 100 plant cell bs water potential increased due to an increase in solutes inside the cell When an organization uses a name, phrase, design, symbols, or a combination of these to identify products and distinguish them from its competitors, it is engaged in Blank______. When marginal costs change depending on the level of activity, a(n) ______ cost behavior pattern exists. The Supreme Court based its 1883 decision about civil rights on an interpretation of someone help me understand this HELPPPPP!! Sarai loves music and decides to paint a music staff on her bedroom wall. She knows that it is important to make the lines parallel, or the end result will not look correct. She pencils in the image below, but before she uses paint, she wants to make sure the lines are parallel. Remembering what she has learned in Geometry class, she uses a piece of tape to make a transversal and decides to measure some angles.Using Sarais measurements, Line 1 must be parallel to Line 3 according to which postulate or theorem?Can you also please explain how you got the answer? thank you